Difference between revisions of "1991 AHSME Problems/Problem 30"
m |
Isabelchen (talk | contribs) m (→Solution 2 (PIE)) |
||
(15 intermediate revisions by 3 users not shown) | |||
Line 3: | Line 3: | ||
For any set <math>S</math>, let <math>|S|</math> denote the number of elements in <math>S</math>, and let <math>n(S)</math> be the number of subsets of <math>S</math>, including the empty set and the set <math>S</math> itself. If <math>A</math>, <math>B</math>, and <math>C</math> are sets for which <math>n(A)+n(B)+n(C)=n(A\cup B\cup C)</math> and <math>|A|=|B|=100</math>, then what is the minimum possible value of <math>|A\cap B\cap C|</math>? | For any set <math>S</math>, let <math>|S|</math> denote the number of elements in <math>S</math>, and let <math>n(S)</math> be the number of subsets of <math>S</math>, including the empty set and the set <math>S</math> itself. If <math>A</math>, <math>B</math>, and <math>C</math> are sets for which <math>n(A)+n(B)+n(C)=n(A\cup B\cup C)</math> and <math>|A|=|B|=100</math>, then what is the minimum possible value of <math>|A\cap B\cap C|</math>? | ||
− | (A) 96 (B) 97 (C) 98 (D) 99 (E) 100 | + | <math>(A) 96 \ (B) 97 \ (C) 98 \ (D) 99 \ (E) 100</math> |
− | == Solution == | + | == Solution 1== |
− | <math>\fbox{}</math> | + | <math>n(A)=n(B)=2^{100}</math>, so <math>n(C)</math> and <math>n(A \cup B \cup C)</math> are integral powers of <math>2</math> <math>\Longrightarrow</math> <math>n(C)=2^{101}</math> and <math>n(A \cup B \cup C)=2^{102}</math>. Let <math>A=\{s_1,s_2,s_3,...,s_{100}\}</math>, <math>B=\{s_3,s_4,s_5,...,s_{102}\}</math>, and <math>C=\{s_1,s_2,s_3,...,s_{k-2},s_{k-1},s_{k+1},s_{k+2},...,s_{100},s_{101},s_{102}\}</math> where <math>s_k \in A \cap B</math> |
+ | Thus, the minimum value of <math>|A\cap B \cap C|</math> is <math>\fbox{B=97}</math> | ||
+ | |||
+ | ==Solution 2 (PIE) == | ||
+ | |||
+ | As <math>|A|=|B|=100</math>, <math>n(A)=n(B)=2^{100}</math> | ||
+ | |||
+ | As <math>n(A)+n(B)+n(C)=n(A \cup B \cup C)</math>, <math>2^{|A|}+2^{|B|}+2^{|C|}=2^{|A \cup B \cup C|}</math>, <math>2^{100}+2^{100}+2^{|C|}=2^{|A \cup B \cup C|}</math> | ||
+ | |||
+ | <math>2^{101}+2^{|C|}=2^{|A \cup B \cup C|}</math> as <math>|C|</math> and <math>|A \cup B \cup C|</math> are integers, <math>|C|=101</math> and <math>|A \cup B \cup C| = 102</math> | ||
+ | |||
+ | By the [[Principle of Inclusion-Exclusion]], <math>|A \cup B| = |A| + |B| - |A \cap B| = 200 - |A \cap B|</math> | ||
+ | |||
+ | <math>|A|=|B| \le |A \cup B| \le |A \cup B \cup C|</math>, <math>100 \le |A \cup B| \le 102</math>, <math>98 \le |A \cap B| \le 100</math> | ||
+ | |||
+ | By the [[Principle of Inclusion-Exclusion]], <math>|A \cup C| = |A| + |C| - |A \cap C| = 201 - |A \cap C|</math> | ||
+ | |||
+ | <math>|C| \le |A \cup C| \le |A \cup B \cup C|</math>, <math>101 \le |A \cup C| \le 102</math>, <math>99 \le |A \cap C| \le 100</math> | ||
+ | |||
+ | By the [[Principle of Inclusion-Exclusion]], <math>|B \cup C| = |B| + |C| - |B \cap C| = 201 - |B \cap C|</math> | ||
+ | |||
+ | <math>|C| \le |B \cup C| \le |A \cup B \cup C|</math>, <math>101 \le |B \cup C| \le 102</math>, <math>99 \le |B \cap C| \le 100</math> | ||
+ | |||
+ | By the [[Principle of Inclusion-Exclusion]], <math>|A \cap B \cap C|=|A \cup B \cup C|- |A| - |B| - |C| + |A \cap B| + |A \cap C|+|B \cap C| = 102-100-100-101+ |A \cap B| + |A \cap C|</math> | ||
+ | <math>+|B \cap C|=|A \cap B| + |A \cap C|+|B \cap C| -199</math> | ||
+ | |||
+ | <cmath>98 + 99 + 99 - 199 \le |A \cap B \cap C| \le 100+100+100-199</cmath> | ||
+ | |||
+ | <cmath>\boxed{\textbf{97}} \le |A \cap B \cap C| \le 101</cmath> | ||
+ | |||
+ | ~[https://artofproblemsolving.com/wiki/index.php/User:Isabelchen isabelchen] | ||
== See also == | == See also == |
Latest revision as of 10:29, 6 May 2023
Problem
For any set , let denote the number of elements in , and let be the number of subsets of , including the empty set and the set itself. If , , and are sets for which and , then what is the minimum possible value of ?
Solution 1
, so and are integral powers of and . Let , , and where Thus, the minimum value of is
Solution 2 (PIE)
As ,
As , ,
as and are integers, and
By the Principle of Inclusion-Exclusion,
, ,
By the Principle of Inclusion-Exclusion,
, ,
By the Principle of Inclusion-Exclusion,
, ,
By the Principle of Inclusion-Exclusion,
See also
1991 AHSME (Problems • Answer Key • Resources) | ||
Preceded by Problem 29 |
Followed by Problem 30 | |
1 • 2 • 3 • 4 • 5 • 6 • 7 • 8 • 9 • 10 • 11 • 12 • 13 • 14 • 15 • 16 • 17 • 18 • 19 • 20 • 21 • 22 • 23 • 24 • 25 • 26 • 27 • 28 • 29 • 30 | ||
All AHSME Problems and Solutions |
The problems on this page are copyrighted by the Mathematical Association of America's American Mathematics Competitions.